This repository has been archived on 2021-03-28. You can view files and clone it, but cannot push or open issues or pull requests.
ICS/hw3/hw3.tex

290 lines
9.8 KiB
TeX

% vim: set ts=2 sw=2 et tw=80:
\documentclass[12pt,a4paper]{article}
\usepackage[utf8]{inputenc} \usepackage[margin=2cm]{geometry}
\usepackage{amstext} \usepackage{amsmath} \usepackage{array}
\newcommand{\lra}{\Leftrightarrow}
\title{Howework 3 -- Introduction to Computational Science}
\author{Claudio Maggioni}
\begin{document} \maketitle
\section*{Question 1}
\[i=1 \hspace{1cm} l_1 = \begin{bmatrix}1\\1\\5\\-3\\\end{bmatrix} \hspace{1cm} u_1 = \begin{bmatrix}
2 & 1 & 1 & -2 \\\end{bmatrix}\]
\[A_2 = \begin{bmatrix}
2 & 1 & 1 &-2 \\ 2 & 2 & -2 & -1 \\ 10 & 4 & 23 & -8 \\
-6 & -2 & 4 & 6 \\\end{bmatrix} - \begin{bmatrix}
2 & 1 & 1 & -2 \\ 2 &1& 1 &-2 \\ 10 & 5 & 5 & -10 \\ -6 & -3 & -3 & 6 \\\end{bmatrix} = \begin{bmatrix}\\& 1 & -3 & 1 \\ & -1 & 18 & 2 \\ & 1 & 7 & 0 \end{bmatrix}\]
\[i = 2 \hspace{1cm} l_2 = \begin{bmatrix}\\1 \\-1 \\1\\\end{bmatrix} \hspace{1cm} u_2 = \begin{bmatrix} & 1& -3& 1\\\end{bmatrix}\]
\[A_3 = \begin{bmatrix}\\& 1 & -3 & 1 \\ & -1 & 18 & 2 \\ & 1 & 7 & 0 \end{bmatrix} - \begin{bmatrix}\\&1 & -3 & 1 \\ & -1 & 3 & -1 \\ & 1 & -3 & 1 \\\end{bmatrix} = \begin{bmatrix}
\\
\\
&&15&3\\&&10&-1\\\end{bmatrix}\]
\[i = 3 \hspace{1cm} l_3 = \begin{bmatrix}\\\\1\\2/3\\\end{bmatrix} \hspace{1cm} u_3 = \begin{bmatrix}&& 15 & 3 \\ \end{bmatrix}\]
\[A_4 = \begin{bmatrix}
\\
\\
&&15&3\\&&10&-1\\\end{bmatrix} - \begin{bmatrix}\\\\&&15 & 3 \\&&10 & 2 \\\end{bmatrix} = \begin{bmatrix}\\\\\\&&&-3\\\end{bmatrix}\]
\[i = 4 \hspace{1cm} l_4 = \begin{bmatrix}\\\\\\1\\\end{bmatrix} \hspace{1cm} u_4 = \begin{bmatrix}&&& -3 \\ \end{bmatrix}\]
\[L = \begin{bmatrix}1\\1&1\\5&-1&1\\-3 &1 &2/3&1\\\end{bmatrix} \hspace{1cm} U = \begin{bmatrix}2 &1 &1&-2\\&1 &-3&1\\&&15&3\\&&&-3\\\end{bmatrix}\]
\[Ly = B \Rightarrow \begin{bmatrix}1\\1&1\\5&-1&1\\-3 &1 &2/3&1\\\end{bmatrix} \begin{bmatrix}y_1\\y_2\\y_3\\y_4\\\end{bmatrix} = \begin{bmatrix}-1\\-3\\36\\18\\\end{bmatrix}
\]
\[y_1 = -1\]
\[y_2 = -3 - (-1) \cdot 1 = -2\]
\[y_3 = 36 - 1 \cdot 2 - (-1) \cdot 5 = 39\]
\[y_4 = 18 - \frac{2}{3} \cdot 39 - (-2) - (-3) \cdot (-1) = -9\]
\[Ux = Y \Rightarrow \begin{bmatrix}2 &1 &1&-2\\&1 &-3&1\\&&15&3\\&&&-3\\\end{bmatrix} \begin{bmatrix}x_1\\x_2\\x_3\\x_4\\\end{bmatrix} = \begin{bmatrix}-1\\-2\\39\\-9\\\end{bmatrix}
\]
\[x_4 = 3\]
\[x_3 = \frac{39 - 3 - 3}{15} = 2\]
\[x_2 = \frac{-2 -3 - (-3) \cdot 2}{1} = 1\]
\[x_1 = \frac{ -1 - (-2) \cdot 3 - 1 \cdot 2 - 1 \cdot 1}{2} = 1\]
\[x = \begin{bmatrix}1\\1\\2\\3\\\end{bmatrix}\]
\section*{Question 2}
\[i = 1 \hspace{1cm} k = 4 \hspace{1cm} \begin{bmatrix}4&2&3&1\\\end{bmatrix}\]
\[
l_1 =
\begin{bmatrix}
1/8 \\
1/4 \\
1/2 \\
1 \\
\end{bmatrix}
\hspace{1cm}
u_1 = \begin{bmatrix} 32 & 24 & 10 & 11\end{bmatrix}
\hspace{1cm}
\]\[
A_2 = \begin{bmatrix}4 &3 &2& 1\\ 8& 8& 5& 2\\ 16& 12& 10& 5\\ 32& 24& 20 &11 \\\end{bmatrix} - \begin{bmatrix}
4 & 3 & 5/2 & 11/8 \\
8 & 6 & 5 & 11/4 \\
16 & 12 & 10 & 11/2 \\
32 & 24 & 20 & 11 \\
\end{bmatrix} =
\begin{bmatrix}
0 & 0 & -1/2 & -3/8 \\
0 & 2 & 0 & -3/4 \\
0 & 0 & 0 & -1/2 \\
0 & 0 & 0 & 0 \\
\end{bmatrix}
\]
\[i = 2 \hspace{1cm} k = 2 \hspace{1cm} p = \begin{bmatrix}4&2&3&1\\\end{bmatrix}\]
\[
l_2 =\begin{bmatrix}
0\\1\\0\\0\\
\end{bmatrix}
\hspace{1cm}
u_2 =\begin{bmatrix} 0 & 2 & 0 & -3/4 \end{bmatrix}\]
\[
A_3 = \begin{bmatrix}
0 & 0 & -1/2 & -3/8 \\
0 & 2 & 0 & -3/4 \\
0 & 0 & 0 & -1/2 \\
0 & 0 & 0 & 0 \\
\end{bmatrix} -
\begin{bmatrix}
0 & 0 & 0 & 0 \\
0 & 2 & 0 & -3/4 \\
0 & 0 & 0 & 0 \\
0 & 0 & 0 & 0 \\
\end{bmatrix}
=
\begin{bmatrix}
0 & 0 & -1/2 & -3/8 \\
0 & 0 & 0 & 0 \\
0 & 0 & 0 & -1/2 \\
0 & 0 & 0 & 0 \\
\end{bmatrix}
\]
\[ i = 3 \hspace{1cm} k = 4 \hspace{1cm} p = \begin{bmatrix}
4&2&1&3\\
\end{bmatrix}
\]
\[
l_3 = \begin{bmatrix}
1 \\0\\0\\0\\
\end{bmatrix}
\hspace{1cm}
u_3 = \begin{bmatrix} 0 & 0& -1/2 & -3/8\\\end{bmatrix}
\]\[
A_4 = \begin{bmatrix}
0 & 0 & -1/2 & -3/8 \\
0 & 0 & 0 & 0 \\
0 & 0 & 0 & -1/2 \\
0 & 0 & 0 & 0 \\
\end{bmatrix} -
\begin{bmatrix}
0 & 0 & -1/2& -3/8\\
0 & 0 & 0 & 0 \\
0 & 0 & 0 & 0 \\
0 & 0 & 0 & 0 \\
\end{bmatrix} =
\begin{bmatrix}
0 & 0 & 0 & 0 \\
0 & 0 & 0 & 0 \\
0 & 0 & 0 & -1/2 \\
0 & 0 & 0 & 0 \\
\end{bmatrix}
\]
\[ i =4 \hspace{1cm} k = 4 \hspace{1cm} p = \begin{bmatrix}
4&2&1&3\\
\end{bmatrix}\]
\[l_4 = \begin{bmatrix} 0 \\0\\1\\0\\\end{bmatrix}
u_4 = \begin{bmatrix}0&0&0&-1/2\end{bmatrix}
\]
\[
P =
\begin{bmatrix}
0 & 0 & 0 & 1 \\
0 & 1 & 0 & 0 \\
1 & 0 & 0 & 0 \\
0 & 0 & 1 & 0 \\
\end{bmatrix}
\]
\[
L = P *
\begin{bmatrix}
1/8 & 0 & 1 & 0 \\
1/4 & 1 & 0 & 0 \\
1/2 & 0 & 0 & 1 \\
1 & 0 & 0 & 0 \\
\end{bmatrix}
=
\begin{bmatrix}
1 & 0 & 0 & 0 \\
1/4 & 1 & 0 & 0 \\
1/8 & 0 & 1 & 0 \\
1/2 & 0 & 0 & 1 \\
\end{bmatrix}
\]
\[
U =
\begin{bmatrix}
32 & 24 & 20 & 11 \\
0 & 2 & 0 & -3/4 \\
0 & 0 & -1/2 & -3/8 \\
0 & 0 & 0 & -1/2 \\
\end{bmatrix}
\]
\section*{Question 4}
\[
A_1 = \begin{bmatrix}
1 & 4 & 8 & 3 \\
4 & 20 & 40 & 28 \\
8 & 40 & 89 & 71 \\
3 & 28 & 71 & 114 \\
\end{bmatrix}
\]
\[ i = 1 \hspace{1cm} l_1 = \begin{bmatrix}1\\4\\8\\3\\\end{bmatrix}\]
\[
A_2 = \begin{bmatrix}
1 & 4 & 8 & 3 \\
4 & 20 & 40 & 28 \\
8 & 40 & 89 & 71 \\
3 & 28 & 71 & 114 \\
\end{bmatrix}
-
\begin{bmatrix}
1 & 4 & 8 & 3\\
4 & 16 & 32 & 12\\
8 & 32 & 64 & 24\\
3 & 12 & 24 & 9\\
\end{bmatrix}
=
\begin{bmatrix}
\\
& 4 & 8 & 16\\
& 8 & 25 & 47 \\
& 16 & 47 & 105 \\
\end{bmatrix}
\]
\[i=2 \hspace{1cm} l_2 = \begin{bmatrix}\\2\\4\\8\\\end{bmatrix}\]
\[A_3 =
\begin{bmatrix}
\\
& 4 & 8 & 16\\
& 8 & 25 & 47 \\
& 16 & 47 & 105 \\
\end{bmatrix}
-
\begin{bmatrix}
\\
& 4 & 8 & 16 \\
& 8 & 16 & 32 \\
& 16 & 32 & 64 \\
\end{bmatrix}
=
\begin{bmatrix}\\\\&& 9 & 15 \\&&15 & 41\\\end{bmatrix}
\]
\[i = 3 \hspace{1cm} l_3 = \begin{bmatrix}\\\\3\\5\\\end{bmatrix}\]
\[A_4 =
\begin{bmatrix}\\\\&& 9 & 15 \\&&15 & 41\\\end{bmatrix}
-
\begin{bmatrix}
\\
\\
&&9&15\\
&& 15 & 25\\
\end{bmatrix}
=
\begin{bmatrix}
\\\\\\&&&16\\
\end{bmatrix}
\]
\[i = 4 \hspace{1cm} l_4 = \begin{bmatrix}\\\\\\4\\\end{bmatrix}\]
\[L = \begin{bmatrix}1\\4&2\\8&4&3\\3&8&5&4\\\end{bmatrix}\]
\section*{Question 5}
\subsection*{Point a)}
First of all, to show that $A_{1,1}$ is symmetric, we say:
\[\begin{bmatrix}A_{1,1} & A_{1,2}\\A_{2,1} & A_{2,2}\\\end{bmatrix} = A = A^T = \begin{bmatrix}A_{1,1}^T & A_{2,1}\\A_{1,2} & A_{2,2}^T\\\end{bmatrix}\]
Therefore we can say that
$$A_{1,1} = A_{1,1}^T$$
and thus $A_{1,1}$ is shown to be symmetric.
Then, since $A$ is positive definite, $x^TAx > 0$ for any vector $x \in R^n$. We choose a vector $x$ such that $x_k = 0$ for any $p < k \leq n$.
Then:
$$ x^TAx = \sum_{j=1}^n x_j \cdot \sum_{i=1}^n x_i a_{i,j} = \sum_{j=1}^p x_j \cdot \sum_{i=1}^n x_i a_{i,j} +
\sum_{j=p+1}^n 0 \cdot \sum_{i=1}^n 0 a_{i,j} =
\sum_{j=1}^p x_j \cdot (\sum_{i=i}^p x_i a_{i,j} +
\sum_{i=p+1}^n 0 a_{i,j}) =$$
$$=\sum_{j=1}^p x_j \cdot \sum_{i=i}^p x_i a_{i,j} =
\begin{bmatrix}x_1 & x_2 & \ldots & x_n\end{bmatrix}
A_{1,1}\begin{bmatrix}x_1 & x_2 & \ldots & x_n\end{bmatrix}^T$$
Then:
$$\begin{bmatrix}x_1 & x_2 & \ldots & x_n\end{bmatrix}A_{1,1}\begin{bmatrix}x_1 & x_2 & \ldots & x_n\end{bmatrix}^T>0$$
which is the definition of positive definiteness for $A_{1,1}$.
\end{document}